Isomorphism between torsion subgroups of $H_q (M)$ and $H_{n-q-1} (M)$ where $M$ is a compact oriented...












2














It it the exercise in Massey's book



Massey, William S., Singular homology theory, Graduate Texts in Mathematics, 70. New York Heidelberg Berlin: Springer- Verlag. XII, 265 p. DM 49.50; $ 29.20 (1980). ZBL0442.55001.



chapter IX, section 5, exercise 5.1, he states that if $M$ is a compact oriented $n$-manifold, then the torsion subgroup of $H_q (M)$ is isomorphic to the torsion subgroup of $H_{n-q-1} (M)$.



I use the fact that a compact oriented $n$-manifold $M$ must have finitely generated homology groups, $mathrm{Hom} (G,mathbb{Q}/mathbb{Z} )$ is the torsion part of $G$ if $G$ is finitely generated and $mathbb{Q} /mathbb{Z} $ is injective to obtain the isomorphism $T(H_q (M))cong H^q (M;mathbb{Q} /mathbb{Z} )$, and use the Poincare duality to give the isomorphism $T(H_q (M))cong H_{n-q} (M;mathbb{Q} /mathbb{Z} )$. However, by the Universal Coefficient Theorem for Tor, I must state that $H_q (M)otimesmathbb{Q} /mathbb{Z} =0$. Will that be true?










share|cite|improve this question






















  • You cannot apply Poincare duality like this because $Bbb Q/Bbb Z$ is not a ring. You need to dualize first, then apply UCT.
    – Mike Miller
    yesterday










  • But Poincare duality can be used here because of theorem 4.1 in this book. I don't find any error on his proof except a lemma for commutativity, it is quite lengthy and I finally choose to believe it.
    – CTW
    yesterday










  • Sorry, you're quite right! (For context, I was thinking of using the product structure on $R$ to take the cap product with the $R$-fundamental class, but you may as well just take cap productwith $Bbb Z$-fundamental class.) Instead the error is in your isomorphism $T(H_q(M)) cong H^q(M;Bbb Q/Bbb Z)$. I will write an answer.
    – Mike Miller
    yesterday
















2














It it the exercise in Massey's book



Massey, William S., Singular homology theory, Graduate Texts in Mathematics, 70. New York Heidelberg Berlin: Springer- Verlag. XII, 265 p. DM 49.50; $ 29.20 (1980). ZBL0442.55001.



chapter IX, section 5, exercise 5.1, he states that if $M$ is a compact oriented $n$-manifold, then the torsion subgroup of $H_q (M)$ is isomorphic to the torsion subgroup of $H_{n-q-1} (M)$.



I use the fact that a compact oriented $n$-manifold $M$ must have finitely generated homology groups, $mathrm{Hom} (G,mathbb{Q}/mathbb{Z} )$ is the torsion part of $G$ if $G$ is finitely generated and $mathbb{Q} /mathbb{Z} $ is injective to obtain the isomorphism $T(H_q (M))cong H^q (M;mathbb{Q} /mathbb{Z} )$, and use the Poincare duality to give the isomorphism $T(H_q (M))cong H_{n-q} (M;mathbb{Q} /mathbb{Z} )$. However, by the Universal Coefficient Theorem for Tor, I must state that $H_q (M)otimesmathbb{Q} /mathbb{Z} =0$. Will that be true?










share|cite|improve this question






















  • You cannot apply Poincare duality like this because $Bbb Q/Bbb Z$ is not a ring. You need to dualize first, then apply UCT.
    – Mike Miller
    yesterday










  • But Poincare duality can be used here because of theorem 4.1 in this book. I don't find any error on his proof except a lemma for commutativity, it is quite lengthy and I finally choose to believe it.
    – CTW
    yesterday










  • Sorry, you're quite right! (For context, I was thinking of using the product structure on $R$ to take the cap product with the $R$-fundamental class, but you may as well just take cap productwith $Bbb Z$-fundamental class.) Instead the error is in your isomorphism $T(H_q(M)) cong H^q(M;Bbb Q/Bbb Z)$. I will write an answer.
    – Mike Miller
    yesterday














2












2








2







It it the exercise in Massey's book



Massey, William S., Singular homology theory, Graduate Texts in Mathematics, 70. New York Heidelberg Berlin: Springer- Verlag. XII, 265 p. DM 49.50; $ 29.20 (1980). ZBL0442.55001.



chapter IX, section 5, exercise 5.1, he states that if $M$ is a compact oriented $n$-manifold, then the torsion subgroup of $H_q (M)$ is isomorphic to the torsion subgroup of $H_{n-q-1} (M)$.



I use the fact that a compact oriented $n$-manifold $M$ must have finitely generated homology groups, $mathrm{Hom} (G,mathbb{Q}/mathbb{Z} )$ is the torsion part of $G$ if $G$ is finitely generated and $mathbb{Q} /mathbb{Z} $ is injective to obtain the isomorphism $T(H_q (M))cong H^q (M;mathbb{Q} /mathbb{Z} )$, and use the Poincare duality to give the isomorphism $T(H_q (M))cong H_{n-q} (M;mathbb{Q} /mathbb{Z} )$. However, by the Universal Coefficient Theorem for Tor, I must state that $H_q (M)otimesmathbb{Q} /mathbb{Z} =0$. Will that be true?










share|cite|improve this question













It it the exercise in Massey's book



Massey, William S., Singular homology theory, Graduate Texts in Mathematics, 70. New York Heidelberg Berlin: Springer- Verlag. XII, 265 p. DM 49.50; $ 29.20 (1980). ZBL0442.55001.



chapter IX, section 5, exercise 5.1, he states that if $M$ is a compact oriented $n$-manifold, then the torsion subgroup of $H_q (M)$ is isomorphic to the torsion subgroup of $H_{n-q-1} (M)$.



I use the fact that a compact oriented $n$-manifold $M$ must have finitely generated homology groups, $mathrm{Hom} (G,mathbb{Q}/mathbb{Z} )$ is the torsion part of $G$ if $G$ is finitely generated and $mathbb{Q} /mathbb{Z} $ is injective to obtain the isomorphism $T(H_q (M))cong H^q (M;mathbb{Q} /mathbb{Z} )$, and use the Poincare duality to give the isomorphism $T(H_q (M))cong H_{n-q} (M;mathbb{Q} /mathbb{Z} )$. However, by the Universal Coefficient Theorem for Tor, I must state that $H_q (M)otimesmathbb{Q} /mathbb{Z} =0$. Will that be true?







algebraic-topology homology-cohomology homological-algebra






share|cite|improve this question













share|cite|improve this question











share|cite|improve this question




share|cite|improve this question










asked yesterday









CTW

838




838












  • You cannot apply Poincare duality like this because $Bbb Q/Bbb Z$ is not a ring. You need to dualize first, then apply UCT.
    – Mike Miller
    yesterday










  • But Poincare duality can be used here because of theorem 4.1 in this book. I don't find any error on his proof except a lemma for commutativity, it is quite lengthy and I finally choose to believe it.
    – CTW
    yesterday










  • Sorry, you're quite right! (For context, I was thinking of using the product structure on $R$ to take the cap product with the $R$-fundamental class, but you may as well just take cap productwith $Bbb Z$-fundamental class.) Instead the error is in your isomorphism $T(H_q(M)) cong H^q(M;Bbb Q/Bbb Z)$. I will write an answer.
    – Mike Miller
    yesterday


















  • You cannot apply Poincare duality like this because $Bbb Q/Bbb Z$ is not a ring. You need to dualize first, then apply UCT.
    – Mike Miller
    yesterday










  • But Poincare duality can be used here because of theorem 4.1 in this book. I don't find any error on his proof except a lemma for commutativity, it is quite lengthy and I finally choose to believe it.
    – CTW
    yesterday










  • Sorry, you're quite right! (For context, I was thinking of using the product structure on $R$ to take the cap product with the $R$-fundamental class, but you may as well just take cap productwith $Bbb Z$-fundamental class.) Instead the error is in your isomorphism $T(H_q(M)) cong H^q(M;Bbb Q/Bbb Z)$. I will write an answer.
    – Mike Miller
    yesterday
















You cannot apply Poincare duality like this because $Bbb Q/Bbb Z$ is not a ring. You need to dualize first, then apply UCT.
– Mike Miller
yesterday




You cannot apply Poincare duality like this because $Bbb Q/Bbb Z$ is not a ring. You need to dualize first, then apply UCT.
– Mike Miller
yesterday












But Poincare duality can be used here because of theorem 4.1 in this book. I don't find any error on his proof except a lemma for commutativity, it is quite lengthy and I finally choose to believe it.
– CTW
yesterday




But Poincare duality can be used here because of theorem 4.1 in this book. I don't find any error on his proof except a lemma for commutativity, it is quite lengthy and I finally choose to believe it.
– CTW
yesterday












Sorry, you're quite right! (For context, I was thinking of using the product structure on $R$ to take the cap product with the $R$-fundamental class, but you may as well just take cap productwith $Bbb Z$-fundamental class.) Instead the error is in your isomorphism $T(H_q(M)) cong H^q(M;Bbb Q/Bbb Z)$. I will write an answer.
– Mike Miller
yesterday




Sorry, you're quite right! (For context, I was thinking of using the product structure on $R$ to take the cap product with the $R$-fundamental class, but you may as well just take cap productwith $Bbb Z$-fundamental class.) Instead the error is in your isomorphism $T(H_q(M)) cong H^q(M;Bbb Q/Bbb Z)$. I will write an answer.
– Mike Miller
yesterday










1 Answer
1






active

oldest

votes


















2














It is not true that $text{Tors}(H_q(M)) cong H^q(M;Bbb Q/Bbb Z)$. You are right that $Bbb Q/Bbb Z$ is injective, so the Ext-term vanishes, but if $H_q(M;Bbb Z) = Bbb Z^{b_q} oplus T$, then $$H^{q}(M;Bbb Q/Bbb Z) cong text{Hom}(H_q(M;Bbb Z), Bbb Q/Bbb Z) cong T oplus (Bbb Q/Bbb Z)^{b_q},$$ noncanonically.



So your approach shows that $T oplus (Bbb Q/Bbb Z)^{b_q} cong H^q(M;Bbb Q/Bbb Z) cong H_{n-q}(M;Bbb Q/Bbb Z)$. What you want to do now is understand the last term. Again we have the universal coefficient sequence,



$$0 to H_{n-q}(M;Bbb Z) otimes Bbb Q/Bbb Z to H_{n-q}(M;Bbb Q/Bbb Z) to text{Tor}(H_{n-q-1}(M;Bbb Z), Bbb Q/Bbb Z) to 0.$$



The first term is the divisible subgroup of $H_{n-q}(M;Bbb Q/Bbb Z)$, and is isomorphic to $(Bbb Q/Bbb Z)^{b_{n-q}} = (Bbb Q/Bbb Z)^{b_q}$. The last term is $text{Tors}(H_{n-q-1}(M;Bbb Z))$.



Because we have an isomorphism of groups $H_{n-q}(M;Bbb Q/Bbb Z) cong H^q(M;Bbb Q/Bbb Z)$, and isomorphisms preserve the maximal divisible subgroup, we see that they preserve the quotient by this subgroup; in both cases it is isomorphic to the finite abelian group $T$.



Note that $H_q(M;Bbb Z) otimes Bbb Q/Bbb Z$ is indeed zero if and only if the claim that $H^q(M;Bbb Q/Bbb Z) cong text{Tors}(H_q(M;Bbb Z))$ is true.






share|cite|improve this answer





















  • Thanks! I have realized of my mistake.
    – CTW
    yesterday










  • @CTW Sure, and thanks for helping me realize my understanding of which coefficients one may take for Poincare duality was incorrect.
    – Mike Miller
    yesterday













Your Answer





StackExchange.ifUsing("editor", function () {
return StackExchange.using("mathjaxEditing", function () {
StackExchange.MarkdownEditor.creationCallbacks.add(function (editor, postfix) {
StackExchange.mathjaxEditing.prepareWmdForMathJax(editor, postfix, [["$", "$"], ["\\(","\\)"]]);
});
});
}, "mathjax-editing");

StackExchange.ready(function() {
var channelOptions = {
tags: "".split(" "),
id: "69"
};
initTagRenderer("".split(" "), "".split(" "), channelOptions);

StackExchange.using("externalEditor", function() {
// Have to fire editor after snippets, if snippets enabled
if (StackExchange.settings.snippets.snippetsEnabled) {
StackExchange.using("snippets", function() {
createEditor();
});
}
else {
createEditor();
}
});

function createEditor() {
StackExchange.prepareEditor({
heartbeatType: 'answer',
autoActivateHeartbeat: false,
convertImagesToLinks: true,
noModals: true,
showLowRepImageUploadWarning: true,
reputationToPostImages: 10,
bindNavPrevention: true,
postfix: "",
imageUploader: {
brandingHtml: "Powered by u003ca class="icon-imgur-white" href="https://imgur.com/"u003eu003c/au003e",
contentPolicyHtml: "User contributions licensed under u003ca href="https://creativecommons.org/licenses/by-sa/3.0/"u003ecc by-sa 3.0 with attribution requiredu003c/au003e u003ca href="https://stackoverflow.com/legal/content-policy"u003e(content policy)u003c/au003e",
allowUrls: true
},
noCode: true, onDemand: true,
discardSelector: ".discard-answer"
,immediatelyShowMarkdownHelp:true
});


}
});














draft saved

draft discarded


















StackExchange.ready(
function () {
StackExchange.openid.initPostLogin('.new-post-login', 'https%3a%2f%2fmath.stackexchange.com%2fquestions%2f3060576%2fisomorphism-between-torsion-subgroups-of-h-q-m-and-h-n-q-1-m-where-m%23new-answer', 'question_page');
}
);

Post as a guest















Required, but never shown

























1 Answer
1






active

oldest

votes








1 Answer
1






active

oldest

votes









active

oldest

votes






active

oldest

votes









2














It is not true that $text{Tors}(H_q(M)) cong H^q(M;Bbb Q/Bbb Z)$. You are right that $Bbb Q/Bbb Z$ is injective, so the Ext-term vanishes, but if $H_q(M;Bbb Z) = Bbb Z^{b_q} oplus T$, then $$H^{q}(M;Bbb Q/Bbb Z) cong text{Hom}(H_q(M;Bbb Z), Bbb Q/Bbb Z) cong T oplus (Bbb Q/Bbb Z)^{b_q},$$ noncanonically.



So your approach shows that $T oplus (Bbb Q/Bbb Z)^{b_q} cong H^q(M;Bbb Q/Bbb Z) cong H_{n-q}(M;Bbb Q/Bbb Z)$. What you want to do now is understand the last term. Again we have the universal coefficient sequence,



$$0 to H_{n-q}(M;Bbb Z) otimes Bbb Q/Bbb Z to H_{n-q}(M;Bbb Q/Bbb Z) to text{Tor}(H_{n-q-1}(M;Bbb Z), Bbb Q/Bbb Z) to 0.$$



The first term is the divisible subgroup of $H_{n-q}(M;Bbb Q/Bbb Z)$, and is isomorphic to $(Bbb Q/Bbb Z)^{b_{n-q}} = (Bbb Q/Bbb Z)^{b_q}$. The last term is $text{Tors}(H_{n-q-1}(M;Bbb Z))$.



Because we have an isomorphism of groups $H_{n-q}(M;Bbb Q/Bbb Z) cong H^q(M;Bbb Q/Bbb Z)$, and isomorphisms preserve the maximal divisible subgroup, we see that they preserve the quotient by this subgroup; in both cases it is isomorphic to the finite abelian group $T$.



Note that $H_q(M;Bbb Z) otimes Bbb Q/Bbb Z$ is indeed zero if and only if the claim that $H^q(M;Bbb Q/Bbb Z) cong text{Tors}(H_q(M;Bbb Z))$ is true.






share|cite|improve this answer





















  • Thanks! I have realized of my mistake.
    – CTW
    yesterday










  • @CTW Sure, and thanks for helping me realize my understanding of which coefficients one may take for Poincare duality was incorrect.
    – Mike Miller
    yesterday


















2














It is not true that $text{Tors}(H_q(M)) cong H^q(M;Bbb Q/Bbb Z)$. You are right that $Bbb Q/Bbb Z$ is injective, so the Ext-term vanishes, but if $H_q(M;Bbb Z) = Bbb Z^{b_q} oplus T$, then $$H^{q}(M;Bbb Q/Bbb Z) cong text{Hom}(H_q(M;Bbb Z), Bbb Q/Bbb Z) cong T oplus (Bbb Q/Bbb Z)^{b_q},$$ noncanonically.



So your approach shows that $T oplus (Bbb Q/Bbb Z)^{b_q} cong H^q(M;Bbb Q/Bbb Z) cong H_{n-q}(M;Bbb Q/Bbb Z)$. What you want to do now is understand the last term. Again we have the universal coefficient sequence,



$$0 to H_{n-q}(M;Bbb Z) otimes Bbb Q/Bbb Z to H_{n-q}(M;Bbb Q/Bbb Z) to text{Tor}(H_{n-q-1}(M;Bbb Z), Bbb Q/Bbb Z) to 0.$$



The first term is the divisible subgroup of $H_{n-q}(M;Bbb Q/Bbb Z)$, and is isomorphic to $(Bbb Q/Bbb Z)^{b_{n-q}} = (Bbb Q/Bbb Z)^{b_q}$. The last term is $text{Tors}(H_{n-q-1}(M;Bbb Z))$.



Because we have an isomorphism of groups $H_{n-q}(M;Bbb Q/Bbb Z) cong H^q(M;Bbb Q/Bbb Z)$, and isomorphisms preserve the maximal divisible subgroup, we see that they preserve the quotient by this subgroup; in both cases it is isomorphic to the finite abelian group $T$.



Note that $H_q(M;Bbb Z) otimes Bbb Q/Bbb Z$ is indeed zero if and only if the claim that $H^q(M;Bbb Q/Bbb Z) cong text{Tors}(H_q(M;Bbb Z))$ is true.






share|cite|improve this answer





















  • Thanks! I have realized of my mistake.
    – CTW
    yesterday










  • @CTW Sure, and thanks for helping me realize my understanding of which coefficients one may take for Poincare duality was incorrect.
    – Mike Miller
    yesterday
















2












2








2






It is not true that $text{Tors}(H_q(M)) cong H^q(M;Bbb Q/Bbb Z)$. You are right that $Bbb Q/Bbb Z$ is injective, so the Ext-term vanishes, but if $H_q(M;Bbb Z) = Bbb Z^{b_q} oplus T$, then $$H^{q}(M;Bbb Q/Bbb Z) cong text{Hom}(H_q(M;Bbb Z), Bbb Q/Bbb Z) cong T oplus (Bbb Q/Bbb Z)^{b_q},$$ noncanonically.



So your approach shows that $T oplus (Bbb Q/Bbb Z)^{b_q} cong H^q(M;Bbb Q/Bbb Z) cong H_{n-q}(M;Bbb Q/Bbb Z)$. What you want to do now is understand the last term. Again we have the universal coefficient sequence,



$$0 to H_{n-q}(M;Bbb Z) otimes Bbb Q/Bbb Z to H_{n-q}(M;Bbb Q/Bbb Z) to text{Tor}(H_{n-q-1}(M;Bbb Z), Bbb Q/Bbb Z) to 0.$$



The first term is the divisible subgroup of $H_{n-q}(M;Bbb Q/Bbb Z)$, and is isomorphic to $(Bbb Q/Bbb Z)^{b_{n-q}} = (Bbb Q/Bbb Z)^{b_q}$. The last term is $text{Tors}(H_{n-q-1}(M;Bbb Z))$.



Because we have an isomorphism of groups $H_{n-q}(M;Bbb Q/Bbb Z) cong H^q(M;Bbb Q/Bbb Z)$, and isomorphisms preserve the maximal divisible subgroup, we see that they preserve the quotient by this subgroup; in both cases it is isomorphic to the finite abelian group $T$.



Note that $H_q(M;Bbb Z) otimes Bbb Q/Bbb Z$ is indeed zero if and only if the claim that $H^q(M;Bbb Q/Bbb Z) cong text{Tors}(H_q(M;Bbb Z))$ is true.






share|cite|improve this answer












It is not true that $text{Tors}(H_q(M)) cong H^q(M;Bbb Q/Bbb Z)$. You are right that $Bbb Q/Bbb Z$ is injective, so the Ext-term vanishes, but if $H_q(M;Bbb Z) = Bbb Z^{b_q} oplus T$, then $$H^{q}(M;Bbb Q/Bbb Z) cong text{Hom}(H_q(M;Bbb Z), Bbb Q/Bbb Z) cong T oplus (Bbb Q/Bbb Z)^{b_q},$$ noncanonically.



So your approach shows that $T oplus (Bbb Q/Bbb Z)^{b_q} cong H^q(M;Bbb Q/Bbb Z) cong H_{n-q}(M;Bbb Q/Bbb Z)$. What you want to do now is understand the last term. Again we have the universal coefficient sequence,



$$0 to H_{n-q}(M;Bbb Z) otimes Bbb Q/Bbb Z to H_{n-q}(M;Bbb Q/Bbb Z) to text{Tor}(H_{n-q-1}(M;Bbb Z), Bbb Q/Bbb Z) to 0.$$



The first term is the divisible subgroup of $H_{n-q}(M;Bbb Q/Bbb Z)$, and is isomorphic to $(Bbb Q/Bbb Z)^{b_{n-q}} = (Bbb Q/Bbb Z)^{b_q}$. The last term is $text{Tors}(H_{n-q-1}(M;Bbb Z))$.



Because we have an isomorphism of groups $H_{n-q}(M;Bbb Q/Bbb Z) cong H^q(M;Bbb Q/Bbb Z)$, and isomorphisms preserve the maximal divisible subgroup, we see that they preserve the quotient by this subgroup; in both cases it is isomorphic to the finite abelian group $T$.



Note that $H_q(M;Bbb Z) otimes Bbb Q/Bbb Z$ is indeed zero if and only if the claim that $H^q(M;Bbb Q/Bbb Z) cong text{Tors}(H_q(M;Bbb Z))$ is true.







share|cite|improve this answer












share|cite|improve this answer



share|cite|improve this answer










answered yesterday









Mike Miller

36.4k470137




36.4k470137












  • Thanks! I have realized of my mistake.
    – CTW
    yesterday










  • @CTW Sure, and thanks for helping me realize my understanding of which coefficients one may take for Poincare duality was incorrect.
    – Mike Miller
    yesterday




















  • Thanks! I have realized of my mistake.
    – CTW
    yesterday










  • @CTW Sure, and thanks for helping me realize my understanding of which coefficients one may take for Poincare duality was incorrect.
    – Mike Miller
    yesterday


















Thanks! I have realized of my mistake.
– CTW
yesterday




Thanks! I have realized of my mistake.
– CTW
yesterday












@CTW Sure, and thanks for helping me realize my understanding of which coefficients one may take for Poincare duality was incorrect.
– Mike Miller
yesterday






@CTW Sure, and thanks for helping me realize my understanding of which coefficients one may take for Poincare duality was incorrect.
– Mike Miller
yesterday




















draft saved

draft discarded




















































Thanks for contributing an answer to Mathematics Stack Exchange!


  • Please be sure to answer the question. Provide details and share your research!

But avoid



  • Asking for help, clarification, or responding to other answers.

  • Making statements based on opinion; back them up with references or personal experience.


Use MathJax to format equations. MathJax reference.


To learn more, see our tips on writing great answers.





Some of your past answers have not been well-received, and you're in danger of being blocked from answering.


Please pay close attention to the following guidance:


  • Please be sure to answer the question. Provide details and share your research!

But avoid



  • Asking for help, clarification, or responding to other answers.

  • Making statements based on opinion; back them up with references or personal experience.


To learn more, see our tips on writing great answers.




draft saved


draft discarded














StackExchange.ready(
function () {
StackExchange.openid.initPostLogin('.new-post-login', 'https%3a%2f%2fmath.stackexchange.com%2fquestions%2f3060576%2fisomorphism-between-torsion-subgroups-of-h-q-m-and-h-n-q-1-m-where-m%23new-answer', 'question_page');
}
);

Post as a guest















Required, but never shown





















































Required, but never shown














Required, but never shown












Required, but never shown







Required, but never shown

































Required, but never shown














Required, but never shown












Required, but never shown







Required, but never shown







Popular posts from this blog

An IMO inspired problem

Management

Has there ever been an instance of an active nuclear power plant within or near a war zone?